返回列表 发帖

想问大家一道题

the programto control the entry of illegal drugs into the country was a failure in 1987. If the program had been successful, the wholesale price of most illegal drugs would not have dropped substansially in 1987.

the assumption in the passage depends on which of the following assumption?

A the supply of illegal drugs dropped substantially  in 1987

B the price paid for most illegal drugs by the average consumer did not drop substantially in 1987

C domestic production of illegal drugs increased at a higher rate than did the entry of such drags into the country

D the wholesale price of a few illegal drugs increased substantially in 1987

E A drop in demond for most illegal drugs in 1987 was not the sole cause of the drop in their wholesale price.
收藏 分享

这题选哪个啊?我猜是D哦!

不知道对否?我的感觉是:这是在问价格的事情,和供求还有生产没有直接关系。比较B和D,B是削弱,而D为正确答案。

如果非法药物能够控制得住的话,那么这种药物的价格就不会下降。言下之意,没有控制得住,因此现在价格很低,D恰好也说这个意思,注意有个词:Few.

TOP

是最后一个哎,我跟你想的一样啊

TOP

返回列表

站长推荐 关闭


美国top10 MBA VIP申请服务

自2003年开始提供 MBA 申请服务以来,保持着90% 以上的成功率,其中Top10 MBA服务成功率更是高达95%


查看